« first day (1412 days earlier)      last day (3073 days later) » 

7:56 AM
I suggested merging two questions in Math Mods' office, quoting more of its duplicates here: but they seem to have interesting (and different) solutions to the problem, can they all be merged to one?
2
Q: If $a_n$ is a strictly increasing unbounded sequence, does $\sum_n \frac{a_{n+1} - a_n}{a_n}$ diverge?

user2566092So I've been thinking through some test cases. If $a_n = n$ then $\sum_n \frac{a_{n+1} - a_n}{a_n}$ is the harmonic series which diverges. And if $a_n = \sum_{k=1}^n 1/k$ then $\sum_n \frac{a_{n+1} - a_n}{a_n}$ diverges like $\sum_n 1/(n \log n)$. So that got me thinking, if $a_n$ is a strictly i...

1
Q: $\sum_{n=1}^\infty \frac{a_{n+1}-a_n}{a_{n+1}}$ diverges

user152715Let $(a_n)_{n\ge 1}$ is an increasing sequence of of positive numbers such that $a_n \to \infty$ as $n \to \infty $. Then $\sum_{n=1}^\infty \frac{a_{n+1}-a_n}{a_{n+1}}$ diverges.

2
Q: A question regarding convergence of a series

AnupamLet $(a_n)$ be an unbounded strictly increasing sequence of positive real numbers and let $x_k=\frac{a_{k+1}-a_{k}}{a_{k+1}}$. Then I want to find the correct option (s). For all $n\geq m, \sum\limits^{n}_{k=m}x_k>1-\frac{m}{n}$ For all $n\geq m, \sum\limits^{n}_{k=m}x_k>\frac{1}{2}$ $\sum\limi...

-2
Q: Convergence of $\sum \limits _{n=1} ^\infty \frac{a_{n+1} - a_n}{a_n}$

MaryamLet $\{a_n\}_{n=1}^\infty$ be a strictly increasing sequence with positive terms; discuss the convergence of the series $\displaystyle\sum_{n=1}^\infty\frac{a_{n+1}-a_n}{a_n}$.

(similar, without unboundedness)
 
 
14 hours later…
9:55 PM
[ SmokeDetector ] Email in answer: Inverse Laplace transform with branch cut by Dale Powers on math.stackexchange.com
 

« first day (1412 days earlier)      last day (3073 days later) »